Happy D's Playland charges a $100 flat
fee for parties plus $1.50 per person.
Marta has $125 to spend. Which
inequality could help Marta find the
number of people she can invite?

Answers

Answer 1

Answer:

$16.66

Step-by-step explanation:

$125-$100 = $25

since its $1.50 per person, we do $25 DIVIDED by $1.50 = $16.66.

i hoped this helped :)


Related Questions

In politics, marketing, etc. We often want to estimate a percentage or proportion p. One calculation in statistical polling is the margin of error - the largest (reasonble) error that the poll could have. For example, a poll result of 72% with a margin of error of 4% indicates that p is most likely to be between 68% and 76% (72% minus 4% to 72% plus 4%). In a (made-up) poll, the proportion of people who like dark chocolate more than milk chocolate was 32% with a margin of error of 2.2%. Describe the conclusion about p using an absolute value inequality.

Answers

Answer: |p-72% |≤ 4%

Step-by-step explanation:

Let p be the population proportion.

The absolute inequality about p using an absolute value inequality.:

[tex]|p-\hat{p}| \leq E[/tex] , where E = margin of error, [tex]\hat{p}[/tex] = sample proportion

Given:  A poll result of 72% with a margin of error of 4% indicates that p is most likely to be between 68% and 76% .

|p-72% |≤ 4%

⇒    72% - 4% ≤ p ≤ 72% +4%

⇒  68%  ≤ p ≤  76%.

i.e. p is most likely to be between 68% and 76% (.

The conclusion about p using an absolute value inequality is in the range of 29.8% to 34.2%.

What is absolute value inequality?

An expression using absolute functions and inequality signs is known as an absolute value inequality.

We know that the absolute value inequality about p using an absolute value inequality is written as,

[tex]|p-\hat p| \leq E[/tex]

where E is the margin of error and [tex]\hat p[/tex] is the sample proportion.

Now, it is given that the poll result of 72% with a margin of error of 4% indicates that p is most likely to be between 68% and 76%. Therefore, p can be written as,

[tex]|p-0.72|\leq 0.04\\\\(0.72-0.04)\leq p \leq (0.72+0.04)\\\\0.68 \leq p\leq 0.76[/tex]

Thus, the p is most likely to be between the range of 68% to 76%.

Similarly, the proportion of people who like dark chocolate more than milk chocolate was 32% with a margin of error of 2.2%. Therefore, p can be written as,

[tex]|p-\hat p|\leq E\\\\|p-0.32|\leq 0.022\\\\(0.32-0.022)\leq p \leq (0.32+0.022)\\\\0.298\leq p\leq 0.342[/tex]

Thus, the p is most likely to be between the range of 29.8% to 34.2%.

Hence, the conclusion about p using an absolute value inequality is in the range of 29.8% to 34.2%.

Learn more about Absolute Value Inequality:

https://brainly.com/question/4688732

Find an equation of the line: Through the point (2, −4) with a y-intercept of −2 Through the points (4,2) and (3,1) Through the point (3,2) with a slope of −2

Answers

Answer and Step-by-step explanation: Equations of line through points and slope can be determined by:

[tex]y-y_{0}=m(x-x_{0})[/tex]

m is slope

Point (2,-4) and y-intercept = -2

Y-intercept is point (0,-2)

m = [tex]\frac{y_{a}-y_{b}}{x_{a}-x_{b}}[/tex]

m = [tex]\frac{-4-(-2)}{2-0}[/tex]

m = - 1

Equation:

[tex]y+2=-1(x-0)[/tex]

[tex]y=-x-2[/tex]

Points (4,2) and (3,1)

m = [tex]\frac{2-1}{4-3}[/tex]

m = 1

Equation:

[tex]y-2=(x-4)[/tex]

[tex]y=x-2[/tex]

Point (3,2) and slope = -2

m = -2

Equation:

[tex]y-2=-2(x-3)[/tex]

[tex]y=-2x+6+2[/tex]

[tex]y=-2x+8[/tex]

If a 100-pound block of ice is placed on an inclined plane that makes an angle of 35° with the horizontal, how much friction force will be required to keep it from sliding down the plane? Choose the equation that could be used to solve the problem if x represents the force required to keep the block from sliding down the plane.

Answers

Answer:

F = 100(.5736)  

= 57.36 lbs. (rounded off to 2 decimal places)  

2) sin60 = .866  

F = 18(.866)  

= 15.59 lbs. (rounded off to 2 decimal places)

Step-by-step explanation:

F = friction

Answer:

100sin35° = x

Step-by-step explanation:

I did the assignment, this was the correct answer for me.

Set A={XIX is an even whole number between 0 and 2) = 0
True? or false?​

Answers

false

Step-by-step explanation:

false

The drama club is selling tickets to its play. An adult ticket costs $15 and a student ticket costs $11. The auditorium will seat 300 ticket-holders. The drama club wants to collect at least $3630 from ticket sales.

Answers

Answer:

83 adult tickets and 217 student tickets.

Step-by-step explanation:

Let number of adult tickets sold = [tex]x[/tex]

Given that total number of tickets = 300

So, number of student tickets = 300 - [tex]x[/tex]

Cost of adult ticket = $15

Cost of student ticket = $11

Total collection from adult tickets = $[tex]15x[/tex]

Total collection from student tickets =  [tex](300-x)\times 11 = 3300-11x[/tex]

Given that overall collection = $3630

[tex]15x+(3300-11x) = 3630\\\Rightarrow 15x-11x=3630-3300\\\Rightarrow 4x = 330\\\Rightarrow x = 82.5[/tex]

So, for atleast $3630 collection, there should be 83 adult tickets and (300-83 = 217 student tickets.

Now , collection = $3632

Find the product of 0.3×0.23

Answers

Answer:

0.069

Step-by-step explanation:

0.3*0.23=0.069

Al’s Produce Stand sells 6 ears of corn for $1.50. Barbara’s Produce Stand sells 13 ears of corn for $3.12. Write two equations, one for each produce stand, that model the relationship between the number of ears of corn sold and the cost.

Answers

Answer:

6n = 1.50

and

13n = 3.12

Step-by-step explanation:

Here in this question, we are interested in writing equations that relate the number of ears of corn sold and the cost.

For Al’s produce stand, let the price per corn sold be n

Thus;

6 * n = 1.50

6n = $1.50 •••••••(i)

For the second;

let the price per corn sold be n;

13 * n = $3.12

-> 13n = 3.12 •••••••••(ii)

write 39/5 as a mixed numer​

Answers

Answer:

6 9/5

Step-by-step explanation:

39/5 as a mixed number;

39/5 as a mixed number;39 ÷ 5 = 6 remaining 9

Therefore:

6 9/5

A shell of mass 8.0-kg leaves the muzzle of a cannon with a horizontal velocity of 600 m/s. Find the recoil velocity of the cannon, if its mass is 500kg.

Answers

Answer:

velocity of recoil velocity of cannon  is -9.6 m/sec

Step-by-step explanation:

according to law of conservation of momentum

total momentum of isolated system of body remains constant.

momentum  = mass of body* velocity of body.

__________________________________

in the problem the system is

shell + cannon

momentum of shell = 8*600 = 4800 Kg-m/sec

let the velocity of cannon be x m/sec

momentum of cannon = 500*x = 500x Kg-m/sec

initially the system of body is in rest (before the shell is fired) hence, total momentum of the system i is 0

applying  conservation of momentum

total momentum before shell fired = total momentum after the shell is fired

0 = momentum of shell + momentum of cannon

4800 + 500x = 0

x = -4800/500 = -9.6

Thus, velocity of recoil velocity of cannon  is -9.6 m/sec

here negative sign implies that direction of velocity of cannon is opposite to that of velocity of shell.

A timeline. 27 B C E to 180 C E PAX ROMANA. 44 B C E The Roman Empire was founded. 80 C E The Colosseum was built. 121 C E Hadrian's Wall was built in England to keep out enemies. 306 C E Constantine became emperor.
How many years passed between the building of the Colosseum and the building of Hadrian’s Wall?

201
121
41
36

Answers

Answer:

the answer is 41

Step-by-step explanation:

C. 41

Step-by-step explanation:

A cube whose edge is 20 cm 1 point
long, has circles on each of its
faces painted black. What is the
total area of the unpainted
surface of the cube if the
circles are of the largest
possible areas?(a) 90.72 cm2 (b)
256.72 cm² (c) 330.3 cm² (d)
514.28 cm?

Answers

Answer:

Unpainted  surface area = 514.28 cm²

Step-by-step explanation:

Given:

Side of cube = 20 Cm

Radius of circle = 20 / 2 = 10 Cm

Find:

Unpainted  surface area

Computation:

Unpainted  surface area = Surface area of cube - 6(Area of circle)

Unpainted  surface area = 6a² - 6[πr²]

Unpainted  surface area = 6[a² - πr²]

Unpainted  surface area = 6[20² - π10²]

Unpainted  surface area = 6[400 - 314.285714]

Unpainted  surface area = 514.28 cm²

A cyclist travels at $20$ kilometers per hour when cycling uphill, $24$ kilometers per hour when cycling on flat ground, and $30$ kilometers per hour when cycling downhill. On a sunny day, they cycle the hilly road from Aopslandia to Beast Island before turning around and cycling back to Aopslandia. What was their average speed during the entire round trip?

Answers

Answer:

Average speed during the trip = 24 km/h

Step-by-step explanation:

Given:

Speed of cyclist uphill, [tex]v_1[/tex] = 20 km/hr

Speed of cyclist on flat ground = 24 km/h

Speed of cyclist downhill, [tex]v_2[/tex] = 30 km/h

Cyclist has traveled on the hilly road to Beast Island from Aopslandia and then back to Aopslandia.

That means, one side the cyclist went uphill will the speed of 20 km/h and then came downhill with the speed of 30 km/h

To find:

Average speed during the entire trip = ?

Solution:

Let the distance between Beast Island and Aopslandia = D km

Let the time taken to reach Beast Island from Aopslandia = [tex]T_1\ hours[/tex]

Formula for speed is given as:

[tex]Speed = \dfrac{Distance}{Time}[/tex]

[tex]v_1 = 20 = \dfrac{D}{T_1}[/tex]

[tex]\Rightarrow T_1 = \dfrac{D}{20} ..... (1)[/tex]

Let the time taken to reach Aopslandia back from Beast Island = [tex]T_2\ hours[/tex]

Formula for speed is given as:

[tex]Speed = \dfrac{Distance}{Time}[/tex]

[tex]v_2 = 30 = \dfrac{D}{T_2}[/tex]

[tex]\Rightarrow T_2 = \dfrac{D}{30} ..... (2)[/tex]

Formula for average speed is given as:

[tex]\text{Average Speed} = \dfrac{\text{Total Distance}}{\text{Total Time Taken}}[/tex]

Here total distance = D + D = 2D km

Total Time is [tex]T_1+T_2[/tex] hours.

Putting the values in the formula and using equations (1) and (2):

[tex]\text{Average Speed} = \dfrac{2D}{T_1+T_2}}\\\Rightarrow \text{Average Speed} = \dfrac{2D}{\dfrac{D}{20}+\dfrac{D}{30}}}\\\Rightarrow \text{Average Speed} = \dfrac{2D}{\dfrac{30D+20D}{20\times 30}}\\\Rightarrow \text{Average Speed} = \dfrac{2D\times 20 \times 30}{{30D+20D}}\\\Rightarrow \text{Average Speed} = \dfrac{1200}{{50}}\\\Rightarrow \bold{\text{Average Speed} = 24\ km/hr}[/tex]

So, Average speed during the trip = 24 km/h

Find the coefficient of third term of (2x−1)^6.

Answers

240

using pascals trianle

for the power 6 it is

1, 6,15,20, 15,6, 1

and for the third term (2x)^4 and (-1)^2

[tex]15 \times {(2x)}^{4} \times {( - 1)}^{2} [/tex]

[tex]240 {x}^{4} [/tex]

Since only the coefficient is needed

the answer is 240.

The required coefficient of third term is 480.


Coefficient of the third term of (2x−1)^6 to be determine.

What is coefficient?

Coefficient is defined as the integer present adjacent to the variable.

Here,  (2x−1)^6
Using binomial expansion,
Third term = P(6,2)(2x)^6-2(-1)^2
                  =   6*5*16x^4
                  = 480x^4

Thus, the required coefficient of third term is 480.


Learn more about coefficient here:

https://brainly.com/question/2507029

#SPJ2

A toy box in the shape of a rectangular prism has a volume of 6,912 cubic inches. The base area of the toy box is 288 square inches. What is the height of the toy box?

Answers

Answer:

h= 24 inches

Step-by-step explanation:

(Volume)= (Base Area) * (Height)

6,912= 288h

h=

20 PTS PLEASE HELP!!!!

Select the correct answer from each drop-down menu.

The function below describes the number of students who enrolled at a university, where f(t) represents the number of students and t represents the time in years.


Initially, (1.03, 3, 19,055, 18,500) students enroll at the university. Every,(1years, t years, 2years, 3years) the number of students who enroll at the university increases by a factor of (1.03, 3, 19,055, 18,500).

Answers

Answer:

Initially 18,500 students

Every 1 year

increase by a factor 1.03

Step-by-step explanation:

The missing information is selected from the given options from the drop down menu. The correct answers are : Initially 18,500 students enroll at the university. Every 1 years the number of students who enroll at the university increases by a factor 1.03.

F(t) = 18,500 * (1.03)^t

Hi how to solve this pythagoras theorem

Answers

Answer:

The perimeter of the triangle is 40.

Step-by-step explanation:

Pythagorean Theorem:  If x and y are the leg lengths of a right triangle, then r = √(x^2 + y^2) is the length of the hypotenuse.  Alternatively, x^2 + y^2 = r^2.

The side lengths 2x, 4x - 1 and 4x + 1 are already arranged in ascending order.  Thus, (2x^)2 + (4x - 1)^2 = (4x + 1).    

Performing the indicated operations, we get:

         4x^2 + 16x^2 - 8x + 1 = 16x^2 + 8x + 1.  Simplify this first by combining like terms:

         20x^2 - 16x = 16x^2, or

         4x^2 - 16x = 0, or

          4x(x - 4) = 0.  Thus, x = 0 (which makes no sense here) or x = 4.  

The perimeter of the rectangle is the sum of the three sides 2x, 4x - 1 and 4x + 1.  Substituting 4 for x, we get

P = 8 + 16 - 1 + 16 + 1, or 40.

The perimeter of the triangle is 40.                        

April typed a 5 page report in 50 mintues. Each page had 500 words at what rate is April typing

Answers

Answer:

Amy types at a rate of 50 words per minute

Step-by-step explanation:

In this question, we are interested in calculating the rate at which April is typing.

From the question, we can deduce that she typed a 5 page report, with each page having a total of 500 words.

Now, if each page has 500 words, the total number of words in all of the pages will be 5 * 500 = 2,500 words

Now, from here, we can see that 2,500 words were typed in 50 minutes.

The number of words per minute will be ;

Total number of words/Time taken = 2500 words/50 minutes

That will give a value of 50 words per minute

which phrase matches the algebraic expression bellow? 2(x+7)+10

Answers

Answer:

i think your answer is two times the sum of x and seven plus ten

if i am wrong than tell me

Step-by-step explanation:

hope this will help :)

Find the total surface area.

Answers

Answer:

143.4 mi²

Step-by-step explanation:

Top: 8x6=48

Bottom: 3x8=24

Sides: 3x8=24 and 24

Trapezoids sides: (6+3)/2*2.6=4.5*2.6=11.7 and 11.7

TOTAL: 48+24+24+24+11.7+11.7= 143.4 mi²

-3 = 7 - BLANK pls tell me what blank is

Answers

Answer:

10

Step-by-step explanation:

-3 = 7 - x

Add x to both sides

x -3 = 7 - x +x

x - 3 = 7

Now, add 3 to both sides

x - 3 + 3 = 7 + 3

x = 10

Answer:

[tex]\boxed{10}[/tex]

Step-by-step explanation:

[tex]-3=7- \sf BLANK[/tex]

[tex]\sf Subtract \ 7 \ from \ sides.[/tex]

[tex]-3-7=-7+7- \sf BLANK[/tex]

[tex]-10=- \sf BLANK[/tex]

[tex]\sf Multiply \ both \ sides \ by \ -1.[/tex]

[tex]-10(-1)=(-1)- \sf BLANK[/tex]

[tex]10= \sf BLANK[/tex]

Need help ASAP!!!! THX

Answers

Answer:

C

Step-by-step explanation:

f(x) = x - 2

f(2) = (2) - 2

f(2) = 0

A + B are wrong cuz..

f(-2) = -2 - 2

f(-2) = -4

If we did not write the equation 5x=21, instead we wrote it 21=5x,
we would get a different solution.
O True
O False

Answers

True we would get a different solution

Answer:

Step-by-step explanation:

5x = 21 and 21 = 5x are identical relationships, and so the solution would be the same in both cases.  (Commutative Property:  order of addition/subtraction is immaterial)

In a given set of items, the mode is items which ?
a. appears first
b. appears fewest
c. appears farthest
d. appears most

Answers

Answer:

d. appears most

Step-by-step explanation:

Mode is the number that appears the most often in a set of data

G(x)= -\dfrac{x^2}{4} + 7g(x)=− 4 x 2 ​ +7g, left parenthesis, x, right parenthesis, equals, minus, start fraction, x, squared, divided by, 4, end fraction, plus, 7 What is the average rate of change of ggg over the interval [-2,4][−2,4]open bracket, minus, 2, comma, 4, close bracket?

Answers

Answer:

-1/2

Step-by-step explanation:

Given the function [tex]G(x)= -\dfrac{x^2}{4} + 7[/tex], the average rate of change of g(x) over the interval [-2,4], is expressed as shown below;

Rate of change of the function is expressed as g(b)-g(a)/b-a

where a - -2 and b = 4

[tex]G(4)= -\dfrac{4^2}{4} + 7\\G(4)= -\dfrac{16}{4} + 7\\G(4)= -4 + 7\\G(4) = 3\\[/tex]

[tex]G(-2) = -\dfrac{(-2)^2}{4} + 7\\G(-2)= -\dfrac{4}{4} + 7\\G(-2)= -1 + 7\\G(-2)= 6[/tex]

average rate of change of g(x) over the interval [-2,4] will be;

[tex]g'(x) = \frac{g(4)-g(-2)}{4-(-2)}\\ g'(x) = \frac{3-6}{6}\\\\g'(x) = -3/6\\g'(x) = -1/2[/tex]

Which of the following choices evaluates (-x)^2 when x=-1
Answers:
1)1
2)-2
3)-1

Answers

(-(-1))^2
(1)^2
1 x 1
=1

Answer: 1

How to do this question plz.
plz work out for me in your notebook or sheet if you can plz the question ​so I can understand more plzz

Answers

Answer:

[tex]3\pi[/tex]

Step-by-step explanation:

The circumference of a circle is [tex]2\pi r[/tex].

If we want to find the circumference of this semi-circle, we can find the circumference if it was a whole circle then divide by 2.

[tex]2 \cdot \pi \cdot r\\2 \cdot \pi \cdot 3\\6 \cdot \pi\\ 6\pi[/tex]

Now we know the circumference of the whole circle.

To find the circumference of half the circle we divide by 2.

[tex]6\pi \div 2 = 3\pi[/tex]

Hope this helped!

Can anyone tell me the answer of the question attached below??

Answers

Answer:  AE = 5

Step-by-step explanation:

I sketched the triangle based on the information provided.

since ∠A = 90° and is divided into three equal angles, then ∠BAD, ∠DAE, and ∠CAE = 30°

Since AB = 5 and BC = 10, then ΔCAB is a 30°-60°-90° triangle which implies that ∠B = 60° and ∠C = 30°

Using the Triangle Sum Theorem, we can conclude that ∠ADB = 90°, ∠ADE = 90°, ∠ AED = 60°, AND ∠ AEC = 120°

We can see that ΔAEC is an isosceles triangle. Draw a perpendicular to divide it into two congruent right triangles. Label the intersection as Z. ΔAEZ and ΔCEZ are 30°-60°-90° triangles.

Using the 30°-60°-90° rules for ΔABC we can calculate that AC = 5√3.

Since we divided ΔAEC into two congruent triangles, then AZ = [tex]\dfrac{5\sqrt 3}{2}[/tex]

Now use the 30°-60°-90° rules to calculate AE = 5

A mother who is 35 years old has two sons, one of whom is twice as old as the other. In 3 years the sum of all their ages will be 59 years. How old are the boys at present ?

Answers

Answer:

son2: 5

son1: 10

Step-by-step explanation:

2x (son1) + x (son2) + 35 (mother) + 3 (years)*3 (people) = 59

3x = 15

x = 5

The age of each boy at present will be 2 years and 3 years.

What is the linear system?

A linear system is one in which the parameter in the equation has a degree of one. It might have one, two, or even more variables.

Let the age of the sons will be x and y.

A mother who is 35 years old has two sons, one of whom is twice as old as the other. Then the equation will be

x = 2y

In 3 years, the sum of all their ages will be 59 years. Then the equation will be

x + y + x + 1 + y + 1 + x + 2 + y + 2 + 35 = 59

Simplify the equation, we have

3x + 3y + 41 = 59

      6y + 3y = 59 – 41

              9y = 18

                y = 2

Then the value of x will be

x = 2y

x = 2(2)

x = 4

Thus, the age of each boy at present will be 2 years and 3 years.

More about the linear system link is given below.

https://brainly.com/question/20379472

#SPJ2

Multiply. (2x - 3)(x + 4) a 2x² + 11x - 12 b 2x² + 5x - 12 c 2x² + 11x - 7 d 2x² + 3x - 7

Answers

Answer:

2x^2 +5x-12

Step-by-step explanation:

(2x - 3)(x + 4)

FOIL

first 2x*x = 2x^2

outer  2x*4 = 8x

inner  -3x

last -3*4 = -12

Add these together

2x^2 +8x-3x-12

Combine like terms

2x^2 +5x-12

Solve this problem... Really urgent ​

Answers

Answer:

[tex] \boxed{\sf Time \ taken = 15 \ minutes} [/tex]

Given:

Initial speed (u) = 65 km/h

Final speed (v) = 85 km/h

Acceleration (a) = 80 km/h²

To Find:

Time taken for car to achieve a speed of 85 km/h in minutes

Step-by-step explanation:

[tex]\sf From \ equation \ of \ motion:[/tex]

[tex] \boxed{ \bold{v = u + at}}[/tex]

By substituting value of v, u & a we get:

[tex] \sf \implies 85 = 65 + 80t[/tex]

Substract 65 from both sides:

[tex] \sf \implies 85 - 65 = 65 - 65 + 80t[/tex]

[tex] \sf \implies 20 = 80t[/tex]

[tex] \sf \implies 80t = 20[/tex]

Dividing both sides by 80:

[tex] \sf \implies \frac{ \cancel{80}t}{ \cancel{80}} = \frac{20}{80} [/tex]

[tex] \sf \implies t = \frac{2 \cancel{0}}{8 \cancel{0}} [/tex]

[tex] \sf \implies t = \frac{ \cancel{2}}{ \cancel{2} \times 4} [/tex]

[tex] \sf \implies t = \frac{1}{4} \: h[/tex]

[tex] \sf \implies t = \frac{1}{4} \times 60 \: minutes[/tex]

[tex] \sf \implies t = 15 \: minutes[/tex]

So,

Time taken for car to achieve a speed of 85 km/h in minutes = 15 minutes

Other Questions
Solve for x: x/25 > 5 Round 18.049 to the nearest tenth. a project will produce cash inflows of 5400 a year for 3 years with a final cash inflow of 2400 in year 4. The projects initial cost is 13400. what is the net present value if the required rate of return is 14.2 percent? Use the map as a reference and select the best answer. In ____ the cataumbo lightning storms last for about 10 hours! La hermana de Sofa es ________. A) Sofa El padre Leonor es ________. B) Letizia La madre Leonor y Sofa es ________. C) Leonor La abuela de Leonor y Sofa es ________. D) Felipe IV Unprecedented industrial growth in the country of Remo has created serious environmental problems because factories there lack adequate pollution control systems. Remo is developing a clean growth plan that includes environmental regulations that will require the installation of such systems. Since no companies in Remo currently produce pollution-control systems, the plan, if implemented, will create significant opportunities for foreign exporters to market pollution-control systemsWhich of the following, if true, most seriously weakens the argument?A. The clean growth plan will provide tax incentives for local business to develop and manufacture pollution-control systemsB. Foreign exporters would provide factory-trained technicians to maintain the pollution-control systems sold in Remo.C. Industrial lobbyists sponsored by local business in Remo are trying to prevent the implementation of the government regulationsD. The regulations that Remo plans to implement are much less strict than those in neighboring nations.E. Pollution in Remo has caused serious health problems for workers, contributing to a significant increase in the number of workdays lost to illness The Constitution states limits on the powers of the national government. What type of governmental system does this reflect In this activity, you will explain global warming: its causes and effects. You may want to pick a certain area to focus on (for example, how global warming will affect sea levels and the fish population).Conduct online research to collect information about the major effects of global warming and climate change on food production. Write a short essay summarizing your research. PLEASE HELP OR JUST GIVE ME AN IDEA F WHAT TO WRITE!!! When you encounter a T-critical value of 2.50, which T-obtained value would reject the null hypothesis? A) Tobt = 2.75 B) Tobt = -2.75 C) Tobt = 2.40 D) Tobt = - 2.40 Wyckam Manufacturing Inc. has provided the following information concerning its manufacturing costs: Fixed Cost per Month Cost per Machine-Hour Direct materials $ 5.40 Direct labor $ 42,400 Supplies $ 0.30 Utilities $ 1,700 $ 0.25 Depreciation $ 15,200 Insurance $ 11,600 For example, utilities should be $1,700 per month plus $0.25 per machine-hour. The company expects to work 4,200 machine-hours in June. Note that the companys direct labor is a fixed cost.Required:Prepare the company's planning budget for manufacturing costs for June. Water discharging into a 10-m-wide rectangular horizontal channel from a sluice gate is observed to have undergone a hydraulic jump. The flow depth and velocity before the jump are 0.8m and 7m/s, respectively. Determine (a) the flow depth and the Froude number after the jump (b) the head loss (c) the dissipation ratio. has a major medical insurance policy with a $500 calendar-year deductible, a $5,000 out-of-pocket limit, and 80-20 coinsurance. was hospitalized for a surgical procedure in March, his first healthcare treatment during the year. The total bill was $20,000. Considering the deductible and coinsurance, how much of this amount must pay Which statement about malaria and sickle-cell anemia is correct? (1 point)Malaria causes a decreased incidence of sickle-cell anemia,Sickle-cell anemia causes an increased incidence of malaria.Sickle-cell anemia is correlated with an increased incidence of malanaMalaria is correlated with a decreased incidence of sickle cell anemia Gavin goes to the market and buys one rectangle shaped board. The length of the board is 16 cm and width of board is 10 cm. If he wants to add a 2 cm wooden border around the board, what will be the area of the rectangle board? Convert from an improper fraction to a mixed number 17\6 Analyze the following scenarios to determine who can appropriately access health information.1. Mrs. John Smith is requesting the emergency room records from last week of her daughter, Katy. Mrs. Smith is the noncustodial parent of Katy, who lives with her dad. Should you release the records to her? Why or why not?2. Mr. Fred Mitchell is requesting the birth record for Amy, his birth daughter. Mr. and Mrs. Mitchell gave Amy up for adoption four years ago. Should you release the records to him? Why or why not?3. Mrs. Lynn Olsen is requesting the lab results of her husband, Tim. She has a note. signed by him, giving his permission for her to have the records. Should you release the records to her? Why or Why not?4. An investigator from the Health and Human Services department is conducting an audit of patient records and has provided a list of records that they want to review. Should you release the information to the investigator? Why or why not?5. Dr. Rex Harrisson is requesting the medical records of Martha Flynn. He states he is a family friend and has been asked by Mrs. Flynn's son to review her last inpatient admission for appropriateness of care. Should you release the records to Dr. Harrison? Why or why not? The width of a rectangle measures (8.3c-8.4d)(8.3c8.4d) centimeters, and its length measures (5.3c+4.8d)(5.3c+4.8d) centimeters. Which expression represents the perimeter, in centimeters, of the rectangle? Emily made a pot cream of pumpkin soup for thanksgiving dinner she put 5 cups of cream in the soup she poured the soup into 24 small bowl show much cream measured in oz is used for each small bowl of soup? which statement is true about this reaction 14/7n + 1/1h ------> 15/8o A. it is a practical source of energy on earth B.it occurs only outside the solar system C.its product is heavier than each of its reactants D.it shows the critical mass of an element The energy change in an endothermic reaction is: A. Internal B. External C. Negative D. Positive